Questions tagged [prime-numbers]

Questions where prime numbers play a key-role, such as: questions on the distribution of prime numbers (twin primes, gaps between primes, Hardy–Littlewood conjectures, etc); questions on prime numbers with special properties (Wieferich prime, Wolstenholme prime, etc.). This tag is often used as a specialized tag in combination with the top-level tag nt.number-theory and (if applicable) analytic-number-theory.

Filter by
Sorted by
Tagged with
0 votes
0 answers
160 views

Is the integer factorization into prime numbers normally distributed?

Let $P_1(n) := 1$ if $n=1$ and $\max_{q|n, \text{ }q\text{ prime}} q$ otherwise, denote the largest prime divisor of $n$. Let us define some rooted trees $T_{n,m}$ for $1 \le m \le n$ by: $T_{n,m}$ ...
mathoverflowUser's user avatar
4 votes
0 answers
107 views

Taking integer values of a sequence of Beurling primes

Let $P=(p_j)_{j=1}^\infty$ be an increasing sequence of real numbers with $1<p_1$ and $\lim_{j\to\infty}p_j=\infty$. As mentioned in [1], Beurling proved that if the multiplicative group $N_P$ ...
Anon12345's user avatar
0 votes
0 answers
222 views

Is parametric formula for congruent numbers worth publishing? [closed]

Earlier in my undergrad career (around 2020) I discovered that all congruent numbers are divisors of Pythagorean triples. Specifically, I can show that they can be written as $$\frac{nm\left(m-n\right)...
ROY BURSON's user avatar
5 votes
2 answers
391 views

On the number of distinct prime factors of $p^2+p+1$

Is it true that, for each positive integer $c$, there exists a prime number $p$ such that $p^2+p+1$ is divisible by at least $c$ distinct primes?
Pablo Spiga's user avatar
8 votes
1 answer
580 views

Arithmetic sequences and Artin's conjecture

(Sorry if this is a naive question; it is not my area!) Consider the following strengthening of Artin's conjecture on primitive roots (and Dirichlet's theorem) for the case of $n=2$: every arithmetic ...
Carl-Fredrik Nyberg Brodda's user avatar
0 votes
1 answer
238 views

Factorization trees and (continued) fractions?

This question is inspired by trying to understand the lexicographic sorting of the natural numbers in the fractal at this question: Is $1 = \sum_{n=1}^{\infty} \frac{\pi(p_n^2)-n+2}{p_n^3-p_n}$ , ...
mathoverflowUser's user avatar
1 vote
0 answers
64 views

Is there an upper bound on the number of partitions of a finite set of primes into 3 sets the products of 2 of which sum to the product of the third?

Is there an upper bound on the number of partitions of a finite set $S$ of prime numbers into 3 sets $A$, $B$ and $C$ for which the following holds?: $$ \prod_{p \in A} p \ + \ \prod_{p \in B} p \ = ...
Stefan Kohl's user avatar
  • 19.3k
4 votes
1 answer
408 views

Smallest prime factor of numbers

The literature refers to smooth integers as \begin{equation}\Psi(x,y):=\#\{n\le x:P_1(n)\le y\},\end{equation} where $P_1(n)$ is the largest prime factor of $n$. There are lots of results studying $\...
alidixon222's user avatar
10 votes
1 answer
294 views

Does the mean ratio of the largest prime factor in prime gaps to the lower bound of the gap converge?

Posting in MO since this questions has been unanswered in MSE for 3 months. Let $p_n$ be the $n$-th prime and $q_n$ be largest among all the prime factors of the composite numbers between $p_n$ and $...
Nilotpal Kanti Sinha's user avatar
2 votes
1 answer
682 views

Is $1 = \sum_{n=1}^{\infty} \frac{\pi(p_n^2)-n+2}{p_n^3-p_n}$ , where $\pi$ denotes the prime counting function and $p_n$ denotes the $n$-th prime?

Is $$1 = \sum_{n=1}^{\infty} \frac{\pi(p_n^2)-n+2}{p_n^3-p_n},$$ where $\pi$ denotes the prime counting function and $p_n$ denotes the $n$-th prime? Context: This question came out as a result in ...
mathoverflowUser's user avatar
-1 votes
0 answers
160 views

Size and type of a Galois radius: are they correlated?

Say as usual that a non negative integer $r$ is a Galois radius of $n$ of type $(a,b)$ if $(n-r,n+r)=(p^a,q^b)$ for some couple of primes $(p,q)$. As primes tend to get sparser as we progress along ...
Sylvain JULIEN's user avatar
4 votes
0 answers
368 views

How to define a fractal from the lexicographic sorting on the prime factorization of natural numbers?

Consider on the natural number the lexicographic ordering on the prime factorization: If $m = p_1^{a_1}\cdots p_r^{a_r},n = q_1^{b_1}\cdots q_s^{b_s}$ then we define: $$m \vartriangleleft n :\iff [(...
mathoverflowUser's user avatar
1 vote
1 answer
198 views

Prime divisors of $p^n-1$, primitive prime divisors

Let $p,q,t_1,t_2$ be distinct prime numbers and let $$k=\frac{p^{qt_1t_2}-1}{p^q-1}.$$ Suppose that $\gcd(k,qt_1t_2)=1$. Is there any reason that $k$ is divisible by at least $7$ distinct prime ...
Pablo Spiga's user avatar
0 votes
1 answer
192 views

A question about the prime counting function

I was playing around with the prime counting function and came across something that seemed correct to me, maybe it's already been proven but I don't know so I decided to ask here. maybe a stupid ...
Egehan Eren's user avatar
1 vote
2 answers
360 views

Solving a recurrence relation for the prime counting function?

I have found some number sequence $c_n = 1+b_n$ for $n \ge 0$, where $b_n = $ A307977(n). I am trying to solve the following recurrence relation for the prime counting function: $$\forall n \ge 3: \pi(...
mathoverflowUser's user avatar
7 votes
2 answers
2k views

Can every integer be written as a sum of squares of primes?

This question is mainly inspired from a different problem I was working on. Is there a value of $k$ such that, for each $n\in \mathbb N$, the equation $$\sum_{i=1}^{k}x_i^2=n$$ is solvable in $x_1,\...
Sayan Dutta's user avatar
4 votes
1 answer
233 views

Density of primes $p$ where $p-1$ has a prime factor exceeding $p^{2/3}$

Fouvry proved* that primes $p$ such that the greatest prime factor, $q$, of $p-1$ is greater than $p^{2/3}$ have positive density in the primes. (The sequence is A073024 in the OEIS.) Are there any ...
Charles's user avatar
  • 8,914
2 votes
0 answers
124 views

Limit of scaled infinite sum with Dirichlet characters modulo 4: is it zero?

I am trying to get an asymptotic formula such as $$ L_4(s, n) \sim L_4(s) + \rho_n(s)\Lambda_n + \frac{\alpha(s)}{\sqrt{n}} + \frac{\beta(s)}{\sqrt{n\log n}}+\cdots$$ where $L_4(s, n)$ is the first $n$...
Vincent Granville's user avatar
3 votes
2 answers
385 views

If $p_1$ and $p_2$ are prime numbers, then either $p_1$ divides $\sum_{i=1}^{p_1-1} i^{p_1p_2-1}$ or $p_2$ divides $\sum_{i=1}^{p_2-1} i^{p_1p_2-1}$?

I feel like it's true as for small cases I couldn't find counterexample. In general, whether it's true that if we have prime number, $p_{1}, p_{2},\dotsc, p_{k}$ and $n=p_{1}p_{2}p_{3}\dotsb p_{k}$ ...
Raj Pratap Singh's user avatar
2 votes
0 answers
112 views

On the integer of the form p^a q^b closest to a given integer N

If we give ourselves a number having only one prime factor $p$ and a given natural integer $N$, we know how to give the integer of the form $p^k$ closest (and less than) to this integer $N$ it's ...
Azoth's user avatar
  • 21
6 votes
2 answers
1k views

Prime gaps within which every "small" prime appears as a factor: Are there only finitely many? Is this the last one?

For a bounded range of positive integers $n,n+1,\ldots,m,$ call a prime number "small" if it does not exceed $\sqrt m,$ so that if one is trying to factor all of these numbers into primes, ...
Michael Hardy's user avatar
2 votes
1 answer
210 views

Prime gaps that are "relatively" bigger than all later prime gaps: Is this in OEIS?

This OEIS entry is about Primes (lower end) with record gaps to the next consecutive prime: primes p(k) where p(k+1) - p(k) exceeds p(j+1) - p(j) for all j < k. I'm wondering about a different ...
Michael Hardy's user avatar
3 votes
0 answers
227 views

The number of continuously increasing primes gaps in the interval $[2,n]$ is less than $\log n$

A prime gap is the difference between two successive prime numbers. The $n$-th prime gap, denoted gn or $g(p_n)$ is the difference between the $(n+1)$-st and the $n$-th prime numbers. Using my ...
Đào Thanh Oai's user avatar
4 votes
1 answer
229 views

Irreducible integral polynomials having roots module primes in arithmetic progressions

Let $f(x)$ be an irreducible polynomial with integer coefficients. One can show (see Exercise 7.2 in this paper of Lenstra) that if $f(x)=0$ has a solution mod $p$ for all but finitely many primes $p$...
Keivan Karai's user avatar
  • 6,014
3 votes
0 answers
264 views

Can you prove and/or generalize this formula involving the Möbius function at n = square free numbers for elliptic curve related sequence in the OEIS?

Let $g(n)$ be the Dirichlet inverse of the Euler totient function: $$g(n) = \sum\limits_{d|n} d \cdot \mu(d)$$ and let $f(x,y)$ be the elliptic equation: $$f(x,y)=x^3 - x^2 - y^2 - y$$ Show that the ...
Mats Granvik's user avatar
  • 1,133
8 votes
2 answers
756 views

Natural density of the set of simple numbers

Let us call $n>1$ simple if every prime power $q$ with $q-1 \mid n-1$ is a prime number. (Please let me know if there is already an established name for these numbers.) The simple numbers $\leq 100$...
Martin Brandenburg's user avatar
13 votes
1 answer
640 views

When is $\mathrm{gcd}(k,p^k-1)=1$ true?

Let $p$ be a prime. Is there a classification of the numbers $k \geq 1$ such that $\gcd(k,p^k-1)=1$? If not, can we at least produce an explicit infinite subset? What is known about these $k$? For the ...
Martin Brandenburg's user avatar
0 votes
1 answer
174 views

Primes above the distant prime neighbors

Let $\ \mathbb P\ $ be the set of all natural primes. Pair $\ (p\ q)\ $ are prime neighbors $\ \Leftarrow:\Rightarrow$ $$ \{x\in\mathbb Z: p\le x\le q\}\cap\mathbb P\,\ =\,\ \{p\,\ q\} $$ Prime $\ x\...
Wlod AA's user avatar
  • 4,674
3 votes
2 answers
395 views

Approximation of partial sum over prime omega function

I asked the question in Math StackExchange. Link: https://math.stackexchange.com/questions/4765476/approximation-of-partial-sum-over-prime-omega-function I haven't got any response yet. Here are the ...
piepie's user avatar
  • 221
2 votes
1 answer
272 views

Factorizations of cyclotomic polynomials valuated at primes

I have a question concerning cyclotomic polynomials valuated at primes. I first state it in the easiest possible form. There exists a function $f:\mathbb{N}\to\mathbb{N}$ such that, if $p$ is a prime, ...
Pablo Spiga's user avatar
2 votes
0 answers
199 views

Not a twin prime pair test using $\gcd$ only

Let $m$ be an odd positive integer such that $m=2k+1$, $k\in\mathbb{N}$. Let $v$ be a vector of $n$ positive integers. Let $v(i)$ be the $i$-th element of the vector. Then we start with $v(i)=m(i+1)-2$...
Notamathematician's user avatar
1 vote
1 answer
301 views

Inequality of three prime factors of $x^2-1$

This is about my experimental math observation on prime factorization of $x^2-1$ We can see, for many $x\in \mathbb{Z}_+$, the expression $x^2-1=(x-1)(x+1)$ gives result as a product of twin prime. ...
Pruthviraj's user avatar
20 votes
3 answers
3k views

What is the simplest proof that the density of coprime pairs does not go to zero?

By density of coprime pairs, I mean the proportion of pairs integers between $1$ and $x$ which are coprime. This is known to be asymptotically $1/\zeta(2)$. I want something much weaker, namely that ...
domotorp's user avatar
  • 18.7k
3 votes
1 answer
277 views

For any integer $n>6$, does there always exist a prime $p>n+1$ such that $p\mid 2^n-1$?

For any integer $n>6$, does there always exist a prime $p>n+1$ such that $p\mid 2^n-1$? It's true for $6<n<100$. But for $n>100$?
hao dong's user avatar
  • 103
4 votes
1 answer
247 views

For any integer $n>0$, does there always exist a prime $p>n$ such that $p\mid 2^n-1$?

For any integer $n>0$, does there always exist a prime $p>n$ such that $p\mid 2^n-1$? It's easy to verify this result for $1<n<100$ by computer. But for any integer $n>0$, is it always ...
hao dong's user avatar
  • 103
1 vote
1 answer
295 views

Some necessary condition for $\gcd(m,n) $ be a proper divisor of $\gcd(mk_2 +nk_1,mn) $ [closed]

Let $m,n,k_1,k_2 $ be natural numbers such that $(k_1,m)=(k_2,n)=1 $. Statement 1: $\gcd(m,n) $ is a proper divisor of $\gcd(mk_2 +nk_1,mn) $, for every $k_1,k_2$ having the above property. Statement ...
Sky's user avatar
  • 913
2 votes
2 answers
387 views

"Squeezing" the primes?

The logical idea here is to map a curve that encodes the primes into the region $(0,1)^2$ and analyze the distribution there more easily and achieve tight bounds. To assess the distribution of primes, ...
53Demonslayer's user avatar
4 votes
0 answers
154 views

On the asymptotic $\pi(x+h(x)) - \pi(x) \sim \frac{h(x)}{\log x} \ (x \to \infty)$

Let $h(x)$ be a function that is positive on $\mathbb{R}_{>0}$ and satisfies $h(x) = o(x)$ and $(\log x)^a = o(h(x))$ for all $a > 0$, as $x \to \infty$. Is it reasonable to expect under these ...
Jesse Elliott's user avatar
0 votes
0 answers
92 views

Question on the inverse Mellin transform $p(x)=\mathcal{M}_s^{-1}\left[-\xi(s)\,\frac{\zeta'(s)}{s\,\zeta(s)^2}\right]\left(\frac{1}{x}\right)$

Consider the function $$p(x)=\underset{K\to\infty}{\text{lim}}\left(\sum\limits_{k=1}^K \Lambda(k) \left(\frac{2 \pi k^2}{x^2}-1\right) e^{-\frac{\pi k^2}{x^2}}\right)\tag{1}$$ where $$P(s)=s\, \...
Steven Clark's user avatar
  • 1,001
-1 votes
1 answer
160 views

An evaluation of the second Chebyshev function

Let $$ \begin{align} \Lambda (n) & &\text{the Von Mangoldt function,}\\ \psi(x)&:=\sum_{n=1}^{[x]}\Lambda (n)&\text{the econd Chebyshev function,}\\ T(x)&:=\sum_{n=1}^{[x]}\log(n). ...
George's user avatar
  • 417
14 votes
1 answer
2k views

Are 0 and 1, respectively, the least and most used digits among primes?

In order to write the first 25 primes (2 to 97), 46 digits are necessary, nine of each of the digits 2, 3, and 7, fewer of all the others. Thereafter, at least for a while, the digit 1 is used more ...
Bernardo Recamán Santos's user avatar
2 votes
0 answers
98 views

On equidistribution of primes in positive characteristic

In S. Lang's book "Algebraic Number Theory" (1986), page 317, Theorem 6 states essentially that given $P$ a set of primes, let $\tau:P\longrightarrow J$ be the typical idèle map taking ...
Hair80's user avatar
  • 655
2 votes
1 answer
684 views

Does the Riemann hypothesis predict a bound for this prime-counting function?

Does the Riemann hypothesis predict an upper bound for $$\left|f(x)-\left(\operatorname{li}(x)-\frac{x}{\log x} \right)\right|,\quad x\ge 2\tag{1}$$ where $$f(x)=\sum\limits_{n=2}^x \frac{\Lambda(n)}{\...
Steven Clark's user avatar
  • 1,001
4 votes
0 answers
243 views

A variant of the Green-Tao theorem

Green and Tao famously proved (The primes contain arbitrarily long arithmetic progressions) that there are arbitrarily long arithmetic progressions in the primes. Specifically, for $k = 3$ this ...
Stanley Yao Xiao's user avatar
1 vote
2 answers
174 views

Prime factors bounded by $k$

Let $S$ be the set of integers with largest prime factor bounded by a given positive integer $k$. Is there a formula for the asymptotic density of such a set $S$?
pallab1234's user avatar
2 votes
1 answer
87 views

Consecutive prime numbers in permutations of digits of the first consecutive positive integers

I have been toying for a while with the study of: in how many distinct primes and of which size can we divide permutations of digits of the first positive integers? In this post I studied how many ...
Juan Moreno's user avatar
9 votes
2 answers
1k views

On the error term of the Riemann explicit formula

Let: $\rho$ be a non-trivial zero of the Riemann zeta function, $\Lambda$ be the von-Mangoldt function and $\psi(x) =\sum_{n \leq x} \Lambda(n)$. What is the best known upper bound for $$f(x, T) := \...
Higgs Boson's user avatar
12 votes
3 answers
3k views

111...11 base p = 111...11 base q

Feels like I am probably missing something obvious. Are there distinct primes $p,q$ and positive integers $m,n$ such that $$ \sum_{i=0}^{n} p^i = \sum_{j=0}^{m} q^j$$ Guessing the answer is no, but ...
Not Bill's user avatar
  • 129
19 votes
2 answers
1k views

Does this number exist?

Does there exist $x\in\mathbb{R}$ such that $\lfloor 10^nx\rfloor$ is a prime number for all $n\in\mathbb{N}$?
Dattier's user avatar
  • 3,609
1 vote
0 answers
85 views

Validity of analysis of summation of function of primes using Abel–Plana summation:

Consider the analytic function $g(x)$ Define $$f(x)=g(x)\frac{\sin^2\left(\frac{π\Gamma(x)}{2x}\right)}{\cos^2\left(\frac{π}{2x}\right)}$$ Note that: $$f(p)=g(p) \text{ for prime } p$$ And $f(n)=0$ ...
TPC's user avatar
  • 690

1
2 3 4 5
41